LSAT and Law School Admissions Forum

Get expert LSAT preparation and law school admissions advice from PowerScore Test Preparation.

 Emily Haney-Caron
PowerScore Staff
  • PowerScore Staff
  • Posts: 577
  • Joined: Jan 12, 2012
|
#45183
Hi rimasu,

That's exactly right. You've got it. :)
 nmaugust
  • Posts: 14
  • Joined: Sep 03, 2018
|
#59759
Is part of the reason that D is correct and not C that

C: says two of the three mergers led to a decline, thus there is still the possibility that one didn't and still leaves room to trust the conclusion?
D: D attacks the "only" by providing another way to gain market share (besides purchasing competitors) in the carpet market: others are dropping out on their own
 Brook Miscoski
PowerScore Staff
  • PowerScore Staff
  • Posts: 418
  • Joined: Sep 13, 2018
|
#61332
nmaugust,

Identifying the conclusion correctly helps you answer this question, and you are on track. The conclusion was that the only way to increase market share was to buy competitors, not marketing. The question stem asks us to weaken the argument.

(D), the correct choice, provides a different way to increase market share--through cost cutting that drives out the competition. When competitors go out of business, that is market share that can be won.

(C) does not tell us anything about market share, so it is irrelevant. This choice is about whether gaining market share is profitable, it is not about how to gain market share.
 leslie7
  • Posts: 73
  • Joined: Oct 06, 2020
|
#83633
I'm just wondering here and I could be wrong but for D to be correct do we not have to assume that producers leaving the market will equate to other companies coming in to take that vacancy?

As in, is that a reasonable assumption to make on the LSAT? (I just don't see this as being very air-tight).

Price reductions, causing others to leave the market could just leave a vaccuum or it could be substituted by another market all together. (Im speculating on the ideas here but my point is that the explanation of a "company can gain market share without purchasing a competitor" attacking the necessary conclusion, we have to assume thats a possibility here no ?
User avatar
 Stephanie Oswalt
PowerScore Staff
  • PowerScore Staff
  • Posts: 804
  • Joined: Jan 11, 2016
|
#83644
leslie7 wrote: Fri Jan 29, 2021 11:13 am I'm just wondering here and I could be wrong but for D to be correct do we not have to assume that producers leaving the market will equate to other companies coming in to take that vacancy?

As in, is that a reasonable assumption to make on the LSAT? (I just don't see this as being very air-tight).

Price reductions, causing others to leave the market could just leave a vaccuum or it could be substituted by another market all together. (Im speculating on the ideas here but my point is that the explanation of a "company can gain market share without purchasing a competitor" attacking the necessary conclusion, we have to assume thats a possibility here no ?
Hi Leslie,

Thanks for the post! I have moved your question to the thread discussing this topic. Please review the full explanation on page 1 and the subsequent discussion, and let us know if you have any additional questions! :)

Thanks!
 leslie7
  • Posts: 73
  • Joined: Oct 06, 2020
|
#83657
Stephanie Turaj wrote: Fri Jan 29, 2021 6:32 pm
leslie7 wrote: Fri Jan 29, 2021 11:13 am I'm just wondering here and I could be wrong but for D to be correct do we not have to assume that producers leaving the market will equate to other companies coming in to take that vacancy?

As in, is that a reasonable assumption to make on the LSAT? (I just don't see this as being very air-tight).

Price reductions, causing others to leave the market could just leave a vaccuum or it could be substituted by another market all together. (Im speculating on the ideas here but my point is that the explanation of a "company can gain market share without purchasing a competitor" attacking the necessary conclusion, we have to assume thats a possibility here no ?
Hi Leslie,

Thanks for the post! I have moved your question to the thread discussing this topic. Please review the full explanation on page 1 and the subsequent discussion, and let us know if you have any additional questions! :)

Thanks!

Hi, thank you for moving the thread to the correct place. I can't recall where I posted this so do you mean to review page one of this thread? If it is this thread I have read the first page and still have the same question. Thank you !
User avatar
 Dave Killoran
PowerScore Staff
  • PowerScore Staff
  • Posts: 5849
  • Joined: Mar 25, 2011
|
#83659
leslie7 wrote: Fri Jan 29, 2021 11:13 am I'm just wondering here and I could be wrong but for D to be correct do we not have to assume that producers leaving the market will equate to other companies coming in to take that vacancy?

As in, is that a reasonable assumption to make on the LSAT? (I just don't see this as being very air-tight).

Price reductions, causing others to leave the market could just leave a vaccuum or it could be substituted by another market all together. (Im speculating on the ideas here but my point is that the explanation of a "company can gain market share without purchasing a competitor" attacking the necessary conclusion, we have to assume thats a possibility here no ?
Hi Leslie,

Thanks for the question! No, you definitely do not have to make that assumption. The conclusion here was that the only way to gain market share was to purchase a competitor. So, looking at it from the standpoint of a hypothetical company, who we will call Acme, according to the author Acme can only gain more market share by buying another company. That's it, that's the only way according to the author. What (D) comes in and says is that there's a different route to getting market share, one that doesn't involve buying a competitor. You can instead drive some current companies out of the market. What happens then? Well, the market share of that driven-out company is then available to be taken by other companies, including Acme. That scenario does not involve relying on the assumption that new companies come into the market at all. But what it does do is suggest a different route than buying a competitor, and that means that buying a competitor is not necessary to gaining market share :)

Please let me know if that helps. Thanks!

Get the most out of your LSAT Prep Plus subscription.

Analyze and track your performance with our Testing and Analytics Package.